If Generations is the third program, then which one of the following could be true?

Jaimee-Salgado on October 3, 2018

Can you please explain?

Can someone please explain the correct answer?

Reply
Create a free account to read and take part in forum discussions.

Already have an account? log in

Mehran on October 16, 2018

@Jaimee-Salgado if G is the third program, it would be at 2pm.

1
2 G G
3

This means T must appear at either 1:30 or 3:30:

1 T
2 G G
3

1
2 G G
3 T

Let's run through the answer choices:

(A) Roamin' is the second program.

This cannot be true because the following scenario would result:

1 R
2 G G
3 S T

This would place W first, which would violate rule 4.

(B) Roamin' is the fifth program.

Violates rule 3 since being fifth would place R last.

(C) Sundown is the fourth program.

Possible as shown here:

1 R T
2 G G
3 S W

So (C) would be the correct answer.

(D) Terry is the fourth program.

Impossible. T must be at either 1:30 (second program) or 3:30 (fifth program).

(E) Waterloo is the second program.

1 W
2 G G
3 T

Violates rule 4 so (E) is eliminated.

Hope that helps! Please let us know if you have any other questions.